B/B UWorld Missed Q's

अब Quizwiz के साथ अपने होमवर्क और परीक्षाओं को एस करें!

A solution of which amino acid will NOT rotate plane polarized light? A) Lysine B) Glycine C) Tyrosine D) Serine

B

Proline differs from the other standard amino acids in that only the former: A) Has a negative charge in its side chain. B) Forms peptide bonds that are planar. C) Contains a secondary amine in its backbone. D) Is commonly found in flexible loop regions of folded proteins

C

Assume that S. typhi immediately enters the bloodstream from the small intestine. Of the following, which would be the first major organ that bloodborne S. typhi would encounter? A) Stomach B) Pancreas C) Large intestine D) Liver

D - The answer to this question is D because blood from the small intestine is transported first to the liver, which regulates nutrient distribution and removes toxins from the blood.

Lysine

Lysine (K) is the only standard amino acid with an amine in its side chain

Agonists vs Antagonists

Membrane-bound receptors respond to ligands that are either endogenous (originating from the organism) or exogenous (foreign to the organism). Agonists are ligands that activate the receptor and promote its downstream effects. In contrast, antagonists are ligands that deactivate or inhibit the receptor, preventing its downstream effects.

Assume L410 is an acquired mutation that occurs only in hepatocytes of adults who were exposed to a certain mutagen. A woman with the L410 mutation has a child with an unaffected man. What is the probability that this child has the L410 mutation? A) 0.00 B) 0.25 C) 0.75 D) 1.00

A - All cells in the body can be classified as either germ (reproductive cells) or somatic (ie, the remaining cells). Germ cells are progenitor cells that undergo meiosis to produce gametes, such as sperm and oocytes. Germline mutations, which occur in the germ cells of parents, can be passed to offspring during sexual reproduction because parental gametic cells combine to form a zygote (fertilized egg). In contrast, parental somatic mutations, which involve alteration of somatic cell lineage (DNA) after conception, will not pass to an offspring because somatic cells are not directly involved in zygote formation. According to the question, the acquired mutation, L410, occurs only in the hepatocytes (liver cells) of adults due to exposure to a mutagen (agent causing genetic mutation). This means that the L410 mutation is somatic because it occurred sporadically in a minority of cells and not in the cells of the entire organism (as would be seen in germline mutations). It can be concluded that L410 occurred during the lifetime of the individual and not in the gametes of his or her parents. Therefore, the somatic L410 mutation in the mother will not be inherited by the offspring, and the probability of this woman and a genetically normal man having a child with the L410 mutation is equal to zero (Choices B, C, and D).

Amino acid catabolism releases nitrogen in the form of ammonia. In the liver, the urea cycle prepares ammonia for excretion. Which amino acid could undergo deamidation to produce ammonia for the urea cycle? A) Glutamine B) Arginine C) Glycine D) Aspartate

A - Arginine, glycine, and aspartate each contain an amino group in their backbones, but they do not have any amido groups in their R-groups. Therefore, these amino acids cannot undergo deamidation.

Which event could alter a protein's primary structure? A) Site-directed mutagenesis of the gene that encodes the protein B) A conformational change in the protein induced by ligand binding C) Protein denaturation by guanidinium chloride D) Reduction of disulfide bonds by β-mercaptoethanol

A - Because primary structure is the sequence of amino acids, it can only be disrupted by events that alter this sequence. These events include cleavage of peptide bonds by proteases and changes in the amino acid sequence by mutations in the DNA that encodes the protein of interest. Of the choices given, only site-directed mutagenesis affects the primary structure.

Which of the following cell-cell junctions would help skin cells withstand mechanical stress by directly anchoring the intermediate filaments of adjacent cells? I Desmosomes II Gap junctions III Tight junctions A) I only B) II only C) I and III only D) II and III only

A - Desmosomes, gap junctions, and tight junctions are cell-cell junctions that provide tissue with mechanical strength, cytoplasmic continuity, and watertight seals, respectively.

The DNA of telomeres can be most accurately described as: A) Noncoding and highly repetitive. B) Single-stranded and self-complementing. C) Able to encode multiple unique proteins. D) Easily cleaved by restriction enzymes

A - Due to a phenomenon called the end-replication problem, the ends of a chromosome cannot be completely replicated, so a chromosome shortens slightly after each replication event. This shortening occurs at the ends of the chromosome, called the telomeres. Telomeres are noncoding DNA sequences that protect the rest of the chromosome from degradation by being slowly degraded themselves. If telomeres encoded proteins, some of the coding information would be lost each time the cell replicated, which would be harmful to the cell. For this reason, telomeres do not encode any proteins

Urinary retention in patients with poorly controlled diabetes mellitus is most likely the result of nerve damage impairing smooth muscle contraction in which of the following structures? A) Bladder B) Urethral sphincters C) Kidney D) Ureter

A - During urination, activity within stretch receptors in the bladder leads to: Contraction of the detrusor muscle, which pushes urine out of the bladder and into the urethra. Relaxation of the IUS, which opens the urethra and allows urine to pass. Nerve damage that impairs the contraction of the detrusor muscle of the bladder would impair emptying of the bladder and lead to urinary retention

Fatty acid oxidation and synthesis occur in which subcellular compartments, respectively? A) In the mitochondria and the cytosol B) In the nucleus and the mitochondria C) In the cytosol and the nucleus D) In the cytosol and the mitochondria

A - Fatty acid oxidation (degradation) and fatty acid synthesis are compartmentalized, with oxidation occurring in the mitochondria and synthesis occurring in the cytosol. The primary metabolic process in the nucleus is the synthesis of nucleic acids (DNA and RNA) from nucleotides.

If lower phospholipid levels in ME patients are due to inhibition of fatty acid synthesis, researchers would most likely observe which of the following? A) Increased cytosolic NADPH. B) Decreased mitochondrial acetyl-CoA. C) Decreased mitochondrial fumarate. D) Increased cytosolic lysine

A - Fatty acid synthesis is an anabolic process that builds lipids in the cytosol. Anabolic processes generally require energy, reducing power, and sufficient precursor molecules: in fatty acid synthesis these requirements are satisfied by ATP, NADPH, and acetyl-CoA, respectively. During fatty acid synthesis, NADPH is oxidized to NADP+ to reduce the carbonyl groups and carbon-carbon double bonds on each acetyl-CoA molecule added to the fatty acid chain. If fatty acid synthesis is inhibited in ME patients, the conversion of NADPH to NADP+ will occur less frequently, and cytosolic NADPH will build up.

Administration of canagliflozin (Administration of a SGLT inhibitor prevents glucose reabsorption) most directly increases urinary output by increasing the: A) Osmotic pressure of the tubular filtrate. B) Osmotic pressure of the peritubular capillaries. C) Hydrostatic pressure of the renal artery. D) Hydrostatic pressure of Bowman's space

A - Hydrostatic pressure and osmotic pressure are two opposing forces that regulate fluid movement between capillaries and the tissues that surround them. Hydrostatic pressure is the force exerted by a liquid on the walls of its container. Accordingly, the hydrostatic pressure of blood, or blood pressure (BP), is the force exerted by blood on the vessel walls. This pressure pushes fluid through capillary wall pores and into the interstitial fluid (the extracellular fluid surrounding cells of a tissue). The kidneys regulate BP by increasing water reabsorption (thereby increasing blood volume and, consequently, BP) when BP is low and by decreasing water reabsorption when BP is high. In contrast, osmotic pressure causes a solution to take in water by osmosis, the passive movement of water molecules from an area of low to an area of high solute concentration. Osmotic pressure increases as solute concentration (osmolarity) increases. When blood osmolarity is high, the kidneys respond by increasing water reabsorption and decreasing solute reabsorption. Conversely, when blood osmolarity is low, the kidneys react by decreasing water reabsorption and increasing solute reabsorption. The passage states that SGLTs transport glucose out of the proximal tubule so it can be returned to circulation through the peritubular capillaries. Administration of a SGLT inhibitor prevents glucose reabsorption, increasing solute concentration within the tubule and consequently increasing the osmotic pressure of the tubular filtrate. As a result, water is drawn back into the renal tubule, increasing urine output.

In a neuron, mitochondrial biogenesis is believed to occur primarily in the cell body, but mitochondria are often positioned at the presynaptic terminal, a distal site with high metabolic demand. Given this information, which molecular mechanism is most likely responsible for mitochondrial transport from the cell body to the presynaptic terminal? A) Kinesin motors transport mitochondria along microtubules B) Kinesin motors transport mitochondria along microfilaments C) Dynein motors transport mitochondria along microfilaments D) Dynein motors transport mitochondria along microtubules

A - Kinesin: Moves intracellular cargo along microtubules in anterograde axonal transport (ie, away from the nucleus and toward distal sites). Dynein: Participates in retrograde axonal transport of intracellular cargo (ie, from distal sites toward the nucleus) (Choice D). According to the question, mitochondria are transported from the cell body toward the presynaptic terminal. Because mitochondria are being transferred from the nucleus and toward the synaptic terminal, this anterograde transport is most likely being performed by kinesin motors moving along the microtubules.

A subset of aggressive cancers has a relatively high growth rate, leading to the formation of large tumors. An effective drug against fast-growing tumors would most likely NOT target which stage of the cell cycle? A) G0 B) G2 C) G2/M checkpoint D) G1/S checkpoint

A - Most cells in the human body are arrested in G0. However, cellular transition into G1 prepares a cell for division and DNA synthesis (S phase). In the G2 phase, DNA is checked for errors and the cell ensures that sufficient organelles and cytoplasm are available for cell division. Subsequently, the cell divides in the M phase via mitosis and cytokinesis. Compounds that inhibit cell division typically target the cell cycle in phases G1 to M.

Which of the following is a potential complication of secondary hyperparathyroidism (a condition characterized by excessive PTH release in response to depressed serum calcium or elevated serum phosphate levels)? A) Decreased bone mineralization B) Decreased osteoclast activity C) Increased osteoblast activity D) Increased cortical bone mass

A - PTH is released in response to decreased plasma calcium and acts on bone, the kidneys, and the small intestine to produce a net increase in calcium and a decrease in phosphate levels (phosphate generally binds calcium to form an insoluble salt, reducing ionized [free] calcium in the blood). The resulting increase in plasma calcium concentration serves as negative feedback and turns off the reflex, ending PTH secretion. Based on the passage, secondary hyperparathyroidism is characterized by increased PTH secretion. PTH release increases bone resorption (breakdown) by osteoclasts, bone-resorbing cells, and causes decreased bone mineralization. The majority of calcium in the body is stored as hydroxyapatite [Ca10(PO4)6(OH)2], a mineral found in the bone matrix that primarily contributes to bone strength and hardness. Osteoclast-mediated bone destruction causes the calcium stored in the matrix to be released into the blood, dissolving mineralized bone and increasing plasma calcium levels. Osteoblasts are bone-forming cells that synthesize the organic and mineralized components of bone. PTH has no direct regulatory effect on osteoblast activity but instead stimulates osteoblasts to secrete cytokines responsible for osteoclast activation.

Which event occurs when a G protein-coupled receptor is activated? A) The α subunit binds GTP. B) The α subunit binds GDP. C) The β subunit binds GTP. D) The β subunit binds GDP.

A - The answer is A because the exchange of GDP for GTP in the α subunit is what accompanies G protein-coupled receptor activation.

Which amino acid has the highest isoelectric focusing point? A) Arg B) Lys C) Asn D) Gln

A - The answer is A because the isoelectric point of an amino acid is the highest for basic amino acids, and arginine and lysine are both basic amino acids. However, arginine has a higher isoelectric point because it is more basic.

Lytic granules are generally released from CTLs when the T-cell receptors on these cells bind specifically to: A) Viral antigens presented on the surface of virus-infected cells. B) Growth factors secreted by helper T lymphocytes. C) B-cell receptors on activated B lymphocytes. D) Constant regions of secreted antibodies.

A - The answer to this question is A because cytotoxic T lymphocytes target virus-infected cells by recognizing the viral antigen presented on the cell surface.

What is the best experimental method to analyze the effect of tdh2 gene deletion on the rate of histone acetylation? Comparing histone acetylation in wild-type and Δtdh2 cells by: A) Western blot B) Southern blot C) Northern blot D) RT-PCR

A - The answer to this question is A because posttranslational modification of proteins such as histone acetylation is analyzed by Western blotting.

The precursor of EGP is translated from a transcript that has had one nontemplated nucleotide added to the open reading frame. This change does not create or eliminate a stop codon. Compared with the protein sGP, which is produced from the unedited transcript, EGP most likely has the same primary: A) Amino-terminal sequence as sGP, but a different primary carboxy-terminal sequence. B) Carboxy-terminal sequence as sGP, but a different primary amino-terminal sequence. C) Sequence as sGP except that EGP has one additional amino acid. D) Sequence as sGP except that EGP has one less amino acid.

A - The answer to this question is A because the addition of one nucleotide to the open reading frame of EGP results in a frameshift mutation and an aberrant carboxy-terminal domain.

In humans, the characteristic tissue of which of the following organs is NOT derived from mesoderm? A) Brain B) Heart C) Kidney D) Skeletal muscle

A - The answer to this question is A because the brain is part of the central nervous system, which is derived from ectoderm. Heart, kidney, and skeletal muscle are derived from mesoderm.

Which series depicts the order in which the precursors of steroid hormones are synthesized? A) Isoprene → monoterpene → squalene → cholesterol B) Cholesterol → isoprene → monoterpene → squalene C) Monoterpene → isoprene → squalene → cholesterol D) Isoprene → cholesterol → monoterpene → squalene

A - The basic unit of cholesterol is a branched five-carbon molecule belonging to a class known as isoprenes. Isoprenes can combine with each other to form larger units known as terpenes. Terpenes consisting of two isoprene units are called monoterpenes, those consisting of two monoterpenes (or four isoprenes) are called diterpenes, and those consisting of three monoterpenes (six isoprenes) are called triterpenes. During cholesterol synthesis, a triterpene known as squalene forms. Squalene undergoes multiple steps, including demethylation, methyl rearrangements, and cyclization, to form cholesterol.

Thermolysin is a protease that can cleave peptides at the N-terminal side of branched alkyl residues. Which of the following peptides has the most thermolysin cleavage sites? A) HQICSVGPKLNT B) DNKGSTVYQRWC C) GHPTLDQIWCER D) RTHCQGYNESDK

A - The question states that thermolysin can cleave peptide bonds at the N-terminal side of branched alkyl residues. Leucine (L), isoleucine (I), and valine (V) are the only amino acids with branched alkyl side chains, and the peptide with the most L, I, and V residues will have the most cleavage sites. The peptide HQICSVGPKLNT has three thermolysin cleavage sites, one each of L, I, and V, and more than any of the other choices given.

Why was Ca2+ added to the oxygen-infused, electrolyte-rich bath? To allow Ca2+ to: A) Be sequestered inside the sarcoplasmic reticulum of the muscle fibers. B) Enter the mitochondria to promote sufficient ATP production by muscle fibers. C) Increase the affinity of muscle fiber myoglobin for oxygen. D) Facilitate the binding of ACh to its receptor on the muscle fiber.

A - The sarcoplasmic reticulum (SR) is a muscle fiber organelle that tightly regulates intracellular calcium (Ca2+) concentration. The release of calcium from the SR promotes muscle fiber contraction whereas the transport of calcium into the SR promotes and maintains muscle fiber relaxation. At the neuromuscular junction, the entry of calcium ions into presynaptic neurons leads to the release of ACh from the presynaptic vesicles and into the synaptic cleft, where ACh then binds its receptor on the muscle fiber membrane. However, because the muscle fibers in the electrolyte bath were completely isolated, no presynaptic neurons were present, and calcium only served to supply the SR of the muscle fibers.

Which amino acid substitution would result in the greatest change in the net charge of a polypeptide at pH 7? A) Glu to Lys B) Asp to Ala C) Lys to Arg D) Arg to Ser

A - The side chain of an amino acid may be positively charged, negatively charged, or neutral, depending on its chemical structure and the ambient pH. At pH 7, the side chains of lysine (Lys) and arginine (Arg) are positively charged whereas those of aspartate (Asp) and glutamate (Glu) are negatively charged. All other amino acids have neutral side chains at pH 7. Removing a negatively charged amino acid (Glu or Asp) from a polypeptide or adding a positively charged one (Arg or Lys) changes the net charge by +1. Conversely, removing a positively charged amino acid or adding a negatively charged one changes the net charge by −1. A Glu-to-Lys mutation removes the negative charge of Glu (change of +1) and adds the positive charge of Lys (change of +1), resulting in a total change of +2. All of the other substitutions listed result in a change with a magnitude of less than 2. Of the choices available, a Glu-to-Lys mutation will result in the greatest change in net charge.

Which of the following procedures would be LEAST likely to prevent bacterial synthesis of the superantigen protein? A) Adding tRNA nucleotides that can bind to mRNA and bacterial ribosomes B) Adding a repressor protein that binds to the operator site of the bacterial superantigen gene C) Adding a specific complementary nucleic acid sequence that can bind to mRNA transcribed from the superantigen gene D) Adding a stop codon within the bacterial superantigen gene

A - The stem asks students to identify the procedure that would be LEAST likely to prevent synthesis of the superantigen. Foils B (binding of the operator), C (binding of the mRNA), and D (insertion of a stop codon) would all be effective ways to prevent synthesis of the superantigen. Adding tRNA nucleotides would be unlikely to prevent synthesis of the antigen.

Stenosis is the abnormal narrowing of a tubular structure in the body, such as a blood vessel. Stenosis of the afferent arterioles in the kidney would be expected to: A) Decrease the glomerular filtration rate. B) Increase blood flow to the glomeruli. C) Decrease resistance in the afferent arterioles. D) Increase output of urine.

A - The volume of fluid filtered through the kidney per unit time is known as the glomerular filtration rate (GFR), which can be modulated by controlling blood flow through the glomerulus. Higher hydrostatic (blood) pressure in the glomerulus increases GFR, and lower hydrostatic pressure decreases GFR.

Secondary hyperparathyroidism may result from: I. Increased intestinal calcium absorption. II. Impaired renal phosphate reabsorption. III. Deficient renal calcitriol synthesis. A) I only B) III only C) I and II only D) I, II, and III

B - Accordingly, PTH acts to increase serum calcium and decrease serum phosphate by: -Stimulating calcitriol synthesis, which then induces absorption of dietary calcium from the small intestine -Promoting bone resorption (osteoclast activity), releasing calcium into the blood -Increasing calcium reabsorption and phosphate excretion in the kidneys The passage explains that secondary hyperparathyroidism involves increased PTH release in response to low serum calcium. Secondary hyperparathyroidism could result from deficient synthesis of renal calcitriol (ie, reduced intestinal absorption of calcium and phosphate), which would lead to low levels of circulating calcium and require constant PTH release to correct this imbalance (Number III).

Individuals undergoing statin therapy would most likely exhibit: A) High levels of leptin. B) Low levels of testosterone. C) Low levels of insulin. D) High levels of cortisol.

B - Common steroid hormones include testosterone, cortisol, aldosterone, estrogen, and progesterone, all of which are secreted by endocrine glands into the bloodstream, where they travel to their target cells. According to the passage, statin therapy treats atherosclerosis by inhibiting the production of a cholesterol precursor. Therefore, statins will decrease the levels of cholesterol and consequently the levels of steroid hormones such as testosterone.

The germ layer that gives rise to the notochord also gives rise to which of the following cell types? A) Alveolar cells B) Osteocytes C) Melanocytes D) Islet beta cells

B - Endoderm (innermost layer): gives rise to accessory digestive organs (eg, liver, pancreas) as well as to the lining (epithelium) of the digestive and respiratory tracts. Ex: mucuous cells, islet beta cells, alveolar cells Mesoderm (middle layer): gives rise to the circulatory system, the musculoskeletal system, and parts of the urinary and reproductive systems. Ex: myocytes (muscle), osteocytes, erythrocytes Ectoderm (outermost layer): gives rise to the nervous system (neurulation) and develops into the integumentary system, which includes hair, skin, nails, and the lining of the mouth, nostrils, and anus. Ex: epidermis, neurons, melanocytes

Surgical removal of the ovaries is one treatment for endometriosis. Complete removal of the ovaries would most likely result in which of the following? A) Significant increase in bone mass B) Degeneration of breast tissue C) Severe immune system impairment D) Increased frequency of menstruation

B - Female sexual development and reproductive function are mediated in large part by the female sex hormones estrogen and progesterone, which are synthesized and secreted by the ovaries (the female gonads). Female sex hormones secreted by the ovaries maintain regular menstrual cycles. After removal of the ovaries, timely secretion of the female sex hormones will not take place and menstrual cycles will not occur.

What observation could have led researchers to conclude that cldnk is expressed in oligodendrocytes? A) A probe made of cldnk mRNA hybridized to RNA in cells that express Sox10. B) An RNA probe that was complementary to cldnk mRNA hybridized in cells that express plp1a. C) A cldnk cDNA probe obtained from oligodendrocytes bound to a microarray with higher affinity than the same probe obtained from other cells. D) A cDNA probe corresponding to cldnk hybridized to the nuclei of oligodendrocytes.

B - For a cell that expresses a gene of interest, one way to determine the cell type is to identify the presence of another mRNA that is known to be specific to the cell type of interest. The passage gives the names of three genes that are specific to oligodendrocytes: plp1a, Sox10, and mbp. If cldnk mRNA is found in the same cells as the mRNA of any of those genes, then cldnk must be expressed in oligodendrocytes. Therefore, if a probe that is complementary to cldnk mRNA hybridizes to cells that alsoexpress plp1A, then cldnk is expressed in oligodendrocytes.

Extended periods of physical exertion in patients with Myalgic encephalopathy (ME), or chronic fatigue syndrome (CFS) result in the conversion of glycine and cysteine to which of the following compounds? A) Fumarate B) Lactate C) Glucose D) Isocitrate

B - Glucogenic amino acids are converted to pyruvate, which can be used to make glucose or citric acid cycle intermediates. Ketogenic amino acids are converted directly to acetyl-CoA, which can enter the citric acid cycle or be used to form ketone bodies. In the absence of oxygen, pyruvate is reduced to lactate to regenerate NAD+ during fermentation. To maintain ATP production, cells switch from aerobic to anaerobic respiration, which does not require oxygen and consists of glycolysis and fermentation. Glycolysis contributes two molecules of ATP by breaking down glucose into two molecules of pyruvate. During fermentation, the NAD+ required for glycolysis is generated by reducing pyruvate to lactate with the enzyme lactate dehydrogenase. Therefore, glycine and cysteine are ultimately converted to lactate during periods of physical exertion.

Glucocorticoid deficiency is known to impair healing of superficial wounds, memory formation, and other physiologic phenomena. This deficiency most likely leads to these wide-ranging effects because: A) Glucose is the primary energy source in many of the body's tissues. B) Receptors for glucocorticoid hormones are expressed in diverse tissue types. C) Glucocorticoids modulate thyroid hormone release by the thyroid gland. D) Hyperplasia of adrenal tissue in CAH causes damage to several nearby organs

B - Hormones affect the function of diverse and distant tissues in the body. This is because hormone receptors can be expressed in a variety of cell types located throughout the body, and any cell that expresses the correct receptor is capable of responding to the receptor's ligand (hormone). For example, glucocorticoids released into the bloodstream from the adrenal cortex influence the energy metabolism of skeletal muscle cells, liver cells, and fat cells, which all express glucocorticoid receptors. However, in glucocorticoid deficiency, an insufficient amount of glucocorticoid hormones in the blood leads to diminished binding of these hormones to their receptors. This condition would compromise the normal function of the various cells that express glucocorticoid receptors.

Iron buildup in the liver can lead to scarring and loss of liver function. If the change in holoTf excretion rate were to cause this form of liver damage, the rate at which holoTf is filtered from the glomerular capillaries must be: A) Decreased due to increased Na+ reabsorption in the nephron. B) Decreased due to decreased hydrostatic pressure within the glomerulus. C) Increased due to increased urinary output from the kidney. D) Increased due to increased hydrostatic pressure within the glomerulus.

B - Hydrostatic blood pressure of arterioles affects glomerular hydrostatic pressure; glomerular hydrostatic pressure pushes fluid out of the glomerular capillaries; capsular hydrostatic pressure pushes fluid into the glomerular capillaries; and plasma osmotic pressure draws fluid into the glomerular capillaries The glomerular filtration rate (GFR) is the rate (amount per unit time) at which fluid is filtered out of the glomerular capillaries (collectively known as the glomerulus) and into Bowman's capsule. GFR is proportional to the hydrostatic (blood) pressure within the glomerulus. As such, GFR can be expected to decrease if the afferent arterioles constrict (ie, because less blood enters the glomerulus) or efferent arterioles dilate (ie, because more blood exits the glomerulus). Decreased glomerular filtration in the kidney will lead to increased retention of iron-bound Tf (holoTf) in the blood, which may contribute to iron buildup in the liver and subsequent organ damage. If hydrostatic pressure within the glomerulus decreases, the GFR (and consequently, the rate of holoTf filtration) will decline because the flow rate of filtered fluid through the kidney will decrease

Based on the sequence that telomerase adds to the ends of chromosomes (5′-TTAGGG-3′), the template portion of TR most likely contains which of the following sequences? A) 5′-UUAGGG-3′ B) 3′-AAUCCC-5′ C) 5′-CCCTAA-3′ D) 3′-CCCUAA-5′

B - In DNA synthesis, the template strand is always complementary to the new strand that is being synthesized. Complementary nucleotide strands always align antiparallel to each other (ie, the 5′ end of one strand aligns with the 3′ end of the other). When the template strand is a DNA molecule, guanine (G) is always paired with cytosine (C) and adenine (A) is always paired with thymine (T). However, when the template strand is an RNA molecule, it will contain uracil (U) instead of thymine. The passage states that TR is the RNA subunit of telomerase that serves as a template for the TERT subunit to lengthen telomeres.

Which of the following structures is derived from the same germ cell layer as the heart? A) Eye B) Bone C) Spinal cord D) Live

B - In gastrulation, the three cell regions, or germ layers, are formed in the embryo. From the outer layer (ectoderm) come cells of the nervous system and epidermis, from the inner layer (endoderm) come cells of the lining of the digestive tube and associated organs, and from the middle layer (mesoderm) come the blood cells, connective tissues (bones, muscles, and tendons), and several organs (kidney, heart, gonads). Thus, the eye and spinal cord are formed from ectoderm, the liver is formed from endoderm, and the heart and bone are formed from mesoderm.

Camels inhabit water-deficient environments and exhibit an evolutionary adaptation that allows them to absorb more water than their ancestors that dwelled in water-rich environments. The diagram below shows the structure of two nephron types present in the kidneys of camels and other mammals. How would the number of cortical nephrons differ from the number of juxtamedullary nephrons in camels and their ancestors? Camels would be expected to have: A) A lower number of juxtamedullary nephrons and a higher number of cortical nephrons compared to their ancestors. B) A higher number of juxtamedullary nephrons and a lower number of cortical nephrons compared to their ancestors. C) A similar number of juxtamedullary nephrons and a lower number of cortical nephrons compared to their ancestors. D) A similar number of juxtamedullary nephrons and a higher number of cortical nephrons compared to their ancestors.

B - In the question diagram, juxtamedullary nephrons have long loops of Henle extending deep into the salty medulla to maximize water reabsorption; in contrast, cortical nephrons have shorter loops of Henle extending shorter distances into the medulla, making them less efficient at water reabsorption. Therefore, compared to their ancestors, camels have likely evolved a higher number of juxtamedullary nephrons and a lower number of cortical nephrons

In neurons, action potential propagation occurs along the axon. The analogous structure in skeletal muscle fibers that enables action potential propagation is the: A) Actin filament. B) Transverse tubule. C) Motor endplate. D) Myelin sheath

B - Muscle fiber action potentials propagate along the sarcolemma, the plasma membrane of the muscle fiber. The sarcolemma burrows deep into the muscle fiber, forming hollow structures known as transverse (T) tubules. Action potentials propagate along T-tubules just as they propagate along the superficial sarcolemma, resulting in the rapid and complete depolarization of the muscle fiber. This promotes calcium release from the sarcoplasmic reticulum, leading to muscle fiber contraction. Actin filaments are one component of the muscle fiber contractile apparatus. However, actin filaments are not involved in the transmission of action potentials.

Transferrin circulating in the blood consists of 679 amino acids, but it is composed of 698 amino acids when it is post-translationally extracted from liver cells, the primary site of transferrin synthesis. Given this information, researchers would most likely be able to extract and isolate the shorter transferrin peptide from: A) Lymph. B) The liquid portion of blood. C) All cellular elements of blood. D) The nucleus of a liver cell.

B - Once secreted from liver cells, the shortened transferrin enters circulation and is incorporated into the plasma. Plasma is the liquid portion of blood, and is made up of water, electrolytes, gases, hormones, nutrients, metabolic waste, and proteins. Although derived from blood, lymph is not a component of blood.

A scientist proposed that phosphorylation of the R domain increases CFTR activity. Which of the following functional groups in the R domain could be removed by mutation to test this hypothesis in vivo? A) Amide B) Hydroxyl C) Thiol D) Carboxylic acid

B - Phosphorylation is a common post-translational modification that is facilitated by protein kinase enzymes. Phosphorylation occurs at serine, threonine, and tyrosine residues, each of which contains a hydroxyl group in its side chain. Protein kinases catalyze the nucleophilic reaction in which the hydroxyl group on the amino acid side chain takes a phosphate group from ATP.

Which reason best explains why reflectin proteins do not adopt a defined tertiary structure? A) Reflectins are unable to interact with water efficiently. B) Reflectins only experience a weak hydrophobic effect. C) Reflectins cannot participate in hydrogen bonding. D) Reflectins are highly basic and therefore cannot fold.

B - Tertiary structure is the three-dimensional folded form of a protein. It is stabilized by multiple forces, including hydrogen bonding and ionic interactions between side chains, but the primary driving force in protein folding is the hydrophobic effect.

Which of the following molecules could decrease the activity of one or more Krebs cycle enzymes? A) Glucagon B) NADH C) Acetyl-CoA D) ADP

B - The Krebs cycle is allosterically regulated at the irreversible steps catalyzed by citrate synthase, isocitrate dehydrogenase, and α-ketoglutarate dehydrogenase. The enzymes are inhibited by NADH, ATP, citrate, and succinyl-CoA, and are activated by ADP and calcium. The Krebs cycle enzymes are not under hormonal control.

Which of the following hormones is LEAST directly regulated by the anterior pituitary? A) Cortisone B) Epinephrine C) Progesterone D) Thyroxin

B - The correct answer is option B, epinephrine. Epinephrine is secreted by the adrenal medulla and its secretion is controlled by the autonomic nervous system. Secretion of the other three hormones is regulated by secretion of anterior pituitary hormones. Cortisone secretion is regulated by adrenocorticotropic hormone (ACTH), progesterone by luteinizing hormone (LH), and thyroxin by thyroid stimulating hormone (TSH).

An infant who experiences seizures and has lactic acid buildup is shown to have deficient pyruvate dehydrogenase complex (PDHC) activity. Genetic analysis shows that both the maternal and paternal alleles for all PDHC subunits are normal. These results could indicate: A) Overexpression of one or more PDHC subunits. B) Impaired ability to synthesize lipoic acid. C) Upregulated acetyl-CoA entry into the Krebs cycle. D) An unusually low rate of pyruvate fermentation.

B - The pyruvate dehydrogenase complex (PDHC) is an enzyme composed of three subunits (E1, E2, and E3). It catalyzes the oxidative decarboxylation of pyruvate to form acetyl-CoA, along with the reduction of NAD+ to form NADH. During this process, electrons are passed from one subunit in the complex to the next until they can be transferred to NAD+. This transfer is facilitated in part by the cofactor lipoic acid. Cofactors are non-amino acid groups of proteins that are necessary for the protein to function. Because they are not amino acids, cofactors are not directly encoded by DNA. The question states that PDHC has decreased activity, yet the alleles (DNA) encoding it are normal. Therefore, some component of PDHC activity that is not directly encoded in the DNA must be altered. Because lipoic acid is a necessary cofactor of the PDHC, impaired lipoic acid production would result in decreased activity of the PDHC complex.

Prolonged starvation in patients diagnosed with hypertriglyceridemia would most likely promote: A) The accumulation of glycerol in the bloodstream. B) The utilization of ketone bodies by the brain. C) The production of glycogen stores in the liver D) The synthesis of amino acids in the liver.

B - Triacylglycerols are esters of glycerol with three fatty acid chains that function as the main energy reserves in animals. These storage lipids are formed in fat cells, or adipocytes, and yield the most ATP when broken down in the liver into glycerol and free fatty acids. Glycerol is rapidly converted to dihydroxyacetone phosphate for glucose synthesis (gluconeogenesis) while free fatty acids enter the mitochondria, where they are oxidized to generate acetyl-CoA for the citric acid cycle. The accumulated acetyl-CoA is converted into ketone bodies such as acetoacetate and 3-hydroxybutyrate. Although the brain primarily uses glucose as a fuel source, it can also use ketone bodies during times of starvation. Based on the passage, patients with hypertriglyceridemia have a surplus of triacylglycerol in their bloodstream. During starvation, excess triacylglycerol in the bloodstream and liver will ultimately be converted into ketone bodies in these patients, and will be utilized by the brain and other tissues.

VEGF signaling also leads to the dilation of existing blood vessels. One function of vasodilation is to: A) Increase body temperature in cold environments. B) Increase blood flow to the intestines following a meal. C) Decrease blood flow to the kidneys during dehydration. D) Maintain blood pressure following an episode of fluid loss.

B - When body temperature is higher than normal (eg, in warm environments), blood vessels supplying the skin vasodilate to facilitate blood flow to the body surface, promoting heat loss through the skin and decreasing body temperature. However, when body temperature is lower than normal (eg, in cold environments), blood vessels supplying the skin vasoconstrict to limit the amount of blood reaching the body surface, limiting heat loss to the environment and increasing body temperature.

Based on the passage, which statement describes Wnt proteins (pi=9)? A) They are composed of multiple subunits. B) They have a positive charge. C) They are synthesized in the smooth endoplasmic reticulum. D) They fold into their tertiary structure in the cytoplasm.

B - Wnt proteins are a family of secretory proteins with isoelectric points around 9, implying that they are positively charged at physiological pH. A is incorrect because there is no information in the passage to support this response. C is incorrect because secretory proteins are synthesized in the rough endoplasmic reticulum. D is incorrect because folding of secretory proteins occurs in the rough endoplasmic reticulum.

Assume a single-celled organism was discovered in water above 100°C. On closer inspection, it is determined that this organism was previously classified under the Archaea domain. Given this information, this organism most likely has: A) A peptidoglycan layer in its cell wall. B) A circular chromosome. C) Telomeres. D) An endoplasmic reticulum

B- Eukaryotic organisms fall under the Eukarya domain, but prokaryotic organisms fall under either the Bacteria or the Archaea domain. In this scenario, the organism is a member of the Archaea domain. Organisms classified under Archaea share traits with bacteria. Like bacteria, archaea are unicellular (single-celled), have a circular chromosome, have no membrane-bound organelles or nucleus, and reproduce asexually via binary fission. However, unlike bacteria, archaea do not have the chemical peptidoglycan in their cell wall

What are the primary myelin-forming cells in the peripheral nervous system? A) Microglia B) Astrocytes C) Schwann cells D) Oligodendrocytes

C

All of the following findings in sperm would cause lower fertilization rates in IVF EXCEPT: A) Defects in microtubule structure B) Mitochondrial deficiency C) Cytoplasmic reduction D) Misfolded acrosomal enzymes

C - A human sperm cell is divided into three segments: a head, a midpiece, and a tail (flagellum). The head contains an acrosome and the nucleus. The acrosome is a flattened structure that encapsulates the tip of the nucleus and is rich in lysosome-like enzymes specialized for piercing the outer shell of an oocyte during fertilization. The midpiece section is packed with mitochondria, essential organelles that produce the ATP required for flagellum-driven sperm motility. This section also contains a pair of central microtubules that are anchored to the cytoskeleton and extend down the length of the flagellum (tail region). The tail, or flagellum, is a singular elongated structure specialized for wavelike movements to propel sperm through a fluid environment. Flagellum-driven motility is derived from the action of ATP-dependent motor proteins that act on the central microtubules. In this experiment, in vitro fertilization (IVF) was used to introduce sperm cells into BCB− and BCB+ oocytes. IVF involves introducing numerous sperm into a dish containing an oocyte. Fully developed sperm generally have little to nonexistent cytoplasm; therefore, reporting that the cytoplasm is reduced in sperm used for IVF would be a normal finding that is unlikely to impact fertilization rate.

For individuals who exhibit insufficient hormone production by the adrenal cortex, ACTH is sometimes administered as a treatment option. Clinicians have reported that most patients fail to respond to this treatment with ACTH. The most likely reason for this outcome is that nonresponsive patients: A) Release insufficient amounts of ACTH from the pituitary gland. B) Continuously overexpress the genes for DBH and PNMT. C) Exhibit impaired cholesterol synthesis in the smooth endoplasmic reticulum of adrenocortical cells. D) Express an overactive ACTH receptor on cells responsive to cortisol.

C - ACTH is released by the anterior pituitary and acts on the adrenal cortex to promote the synthesis and secretion of cortisol, a steroid hormone produced from cholesterol. However, the actual synthesis of this steroid hormone depends on the production of cholesterol by SER proteins, not ACTH. Consequently, defective SER proteins may compromise cortisol synthesis within the adrenal cortex regardless of ACTH stimulation. Accordingly, patients with adrenal insufficiency caused by defective SER proteins would not respond to ACTH administration as a treatment.

In the DNA polymerase active site, which nucleotide base would provide the most Watson-Crick donors for interactions with amino acid residues? A) Adenine B) Cytosine C) Guanine D) Thymine

C - Adenine (A) and thymine (T), which each have 1 donor and 1 acceptor atom, form two hydrogen bonds between them (note that thymine has two potential acceptors in the form of carbonyls, but only one participates in Watson-Crick pairing). Guanine (G) and cytosine (C) pairs form three hydrogen bonds: Guanine contributes 1 acceptor and 2 donors whereas cytosine contributes 2 acceptors and 1 donor.

Human cardiac muscle differs from skeletal muscle because it: A) Appears striated. B) Requires acetylcholine to contract. C) Contains intercalated discs. D) Contains cells with multiple nuclei.

C - Cardiac muscle is unique in that each cell is connected to adjacent cells via intercalated discs, which are regions of cell contact that contain both desmosomes (to prevent cells from separating during contraction) and gap junctions (to facilitate direct ion exchange for synchronized contraction).

Scientists could confirm that an allosteric effector increases the catalytic efficiency of an enzyme if it has what effect on kcat and Km? A) kcat decreases, Km increases B) kcat decreases, Km remains unchanged C) kcat increases, Km decreases D) kcat remains unchanged, Km increases

C - Catalytic efficiency is a measure of how well an enzyme facilitates reactions at low substrate concentrations. When enzyme concentration is held constant, catalytic efficiency is proportional to the initial slope of a Michaelis-Menten curve (when [S] << Km). This slope is governed by both the catalytic turnover kcat and the Michaelis constant Km. Mathematically, catalytic efficiency is calculated as the ratio of kcat to Km. An increase in kcat increases the catalytic efficiency, and a decrease in Km also increases the efficiency. Therefore, an allosteric activator that both increases kcat and decreases Km will increase the catalytic efficiency of an enzyme. Conceptually, a large (steep) slope indicates an efficient enzyme because a small amount of substrate yields a high reaction rate. Catalytic turnover (kcat) measures the number of substrate molecules converted to product per enzyme per second when the enzyme is saturated; therefore, a high kcat corresponds to a high maximum rate (Vmax) and, by extension, a larger slope at lower substrate concentrations. The Michaelis constant (Km) is an indicator of binding affinity. A small Km indicates high affinity, allowing the reaction to approach Vmax more rapidly (higher slope) with smaller amounts of substrate. Therefore, a small Km contributes to high catalytic efficiency.

A scientist proposed that ectopic endometrial cells contain extra copies of the VEGF gene. Given this information, which of the following techniques can be used to analyze VEGF gene count? I DNA sequencing II Northern blot III Southern blot A) I only B) II only C) I and III only D) II and III only

C - DNA sequencing and Southern blotting are DNA assays that may be used to assess the relative quantity of genes between tissue types. Northern blotting is an RNA assay used to assess gene expression in different tissues. (you got this right, just want to make sure you remember all the different techniques)

Which factor decreases the stability of a DNA double helix during denaturation? A) Base stacking is enhanced. B) Hydrophobic effects are increased. C) Hydrogen bonds are disrupted. D) Phosphodiester bonds are broken.

C - During DNA double helix formation, hydrogen bonds form between the bases and release energy. Heat energy provided in a DNA melting reaction is used to disrupt interactions that contribute to DNA double helix structure: hydrogen bonding, base stacking, and hydrophobic effects. Covalent phosphodiester bonds of the sugar-phosphate backbone maintain the structure of single-stranded DNA. These linkages are not affected when the double helix is separated during denaturation.

In individuals with glycosuria (excretion of glucose in the urine), glucose that is not reabsorbed in the proximal tubule next flows into which of the following nephron structures? A) The distal tubule B) The collecting duct C) The loop of Henle D) Bowman's capsule

C - Each nephron consists of a glomerulus (a bed of capillaries that receives blood from the renal arteries) and a long tubule where filtered fluid is concentrated into urine. Blood pressure forces fluid in the blood through the porous glomerular capillaries into Bowman's capsule (Choice D), a cup-shaped extension of the renal tubule that surrounds the glomerulus. This fluid (now called filtrate) contains small solutes such as glucose, ions, and urea but lacks blood cells and proteins, which remain in circulation. The filtrate passes from Bowman's capsule into a long tubule, comprised successively of the following segments: Proximal tubule: In this segment, important nutrients (eg, amino acids, vitamins, salts, glucose, water) are reabsorbed from the nephron and incorporated back into the blood through the peritubular capillaries that surround the tubule. Waste products not filtered by Bowman's capsule are actively secreted from the peritubular capillaries into the nephron. The loop of Henle (Choice C): This segment is a hairpin structure composed of two limbs. The descending limb extends down into the relatively salty medulla (inner portion of the kidney) and is permeable to water, allowing the passive reabsorption of water via osmosis. The ascending limb transports filtrate out of the medulla and back to the cortex (outer portion of the kidney). This limb is permeable to salt but impermeable to water; this leads to active reabsorption of ions only, which concentrates the medullary interstitial fluid (maintains its saltiness) and prevents its dilution. Distal tubule (Choice A): Antidiuretic hormone (ADH; vasopressin) and aldosterone promote the reabsorption of water in this segment of the tubule. Additional waste products are secreted into the nephron from the peritubular capillaries. Collecting duct (Choice B): ADH and aldosterone act on the collecting duct in the same way they act on the distal tubule. The now-concentrated urine is emptied into the ureters for excretion.

During a parasitic infection, which of the following would most likely be responsible for phagocytosis of organisms and the release of histamine, respectively? A) Basophils, macrophages B) Natural killer cells, T-cells C) Macrophages, basophils D) T-cells, natural killer cells

C - Macrophages are one of the cells responsible for phagocytosis, the process of engulfing solid particles (eg, bacteria) designated for destruction into phagocytic vesicles called phagosomes. In contrast, basophils function to release chemical mediators such as histamine that stimulate an inflammatory response.

If a viral antigen were to bind to the receptors on the surface of a B lymphocyte, which of the following immune responses would NOT occur? A) Division of the B lymphocyte into cells that differentiate to secrete antibodies B) Interaction of the B lymphocyte with other immune cells that stimulate B-lymphocyte proliferation C) Secretion of toxins by the B lymphocyte that destroy nearby virus-infected cells D) Proliferation of cells that can recognize the antigen more rapidly in the event of a future infection

C - Natural killer and cytotoxic T cells, not B lymphocytes, respond to antigens by releasing toxins that induce apoptosis (cell death) in nearby infected cells. Humoral immunity is an antibody-mediated immune response. During an infection, B lymphocytes bind foreign antigens and become activated. Activated lymphocytes divide into many identical cells, some of which differentiate into short-lived plasma cells that secrete antibodies during the immediate immune response. The remaining cells differentiate into long-lived memory cells, which can respond more rapidly to future infections.

During protein folding, which of the following interactions help stabilize secondary, tertiary, or quaternary structure? I. Hydrogen bonds II. Ionic bonds III. van der Waals forces IV. Amide bonds A) I and II only B) III and IV only C) I, II, and III only D) I, III, and IV only

C - Secondary structure is driven and reinforced by hydrogen bonding between a carbonyl oxygen and an amino hydrogen of nearby amino acids. Hydrogen bonds bend the polypeptide into α-helices and β-pleated sheets, the main components of secondary structure. Tertiary structure is formed by the folding of α-helices and β-pleated sheets into a three-dimensional structure. Although this folding is driven by hydrophilic and hydrophobic interactions, the resulting structure is reinforced by van der Waals forces, hydrogen bonding between side chains, and ionic bonds (electrostatic interactions or "salt bridges") formed between acidic and basic side chains. Tertiary structure may also be reinforced by disulfide bonds (S-S), which are covalent bonds between thiol groups (-SH) of 2 cysteine side chains. Quaternary structure refers to the assembly or 2 or more polypeptide chains (subunits) and is reinforced by the same chemical interactions as tertiary structure. (Number IV) The primary structure of a protein (linear sequence of amino acids) is maintained by amide bonds (ie, peptide bonds) that link the amino acids into a polypeptide chain. After denaturation, the primary structure remains intact; therefore, renaturation would not require reinstatement of amide bonds.

The systemic blood pressure gradient ΔP is equal to the mean pressure of blood exiting the heart through arteries minus the mean pressure of blood returning to the heart through veins. The relationship of ΔP to cardiac output (CO) and vascular resistance (VR) is given by: ΔP=CO×VR Given this, which of the following physiological changes would cause increased VR within vessels of the circulatory system? (Note: Assume all other physiological factors remain constant.) A) Increased vasodilation of arteries B) Heightened activity of the SA node C) Reduced ventricular filling prior to contraction D) Decreased arterial blood pressure

C - Stroke volume depends on the amount of blood loaded into the left ventricle prior to contraction. Reduced ventricular filling would lead to less blood being expelled from the ventricle with each contraction and decreased CO. Given the relationship of VR to CO, decreased CO (in the absence of other physiological changes) would ultimately increase VR.

How many molecules of NADH are produced from six molecules of glucose that undergo glycolysis? A) 3 B) 6 C) 12 D) 18

C - The answer is C because each cycle of glycolysis produces two NADH molecules and requires an input of one glucose molecule.

Which example of movement across a cell membrane is an example of active transport? A) Water moving through aquaporins B) Potassium moving through an ion channel C) Sodium and potassium moving through the Na+K+ ATPase D) Chloride and bicarbonate moving through the chloride—biocarbonate exchanger

C - The answer is C because the Na+K+ ATPase uses the energy of ATP hydrolysis to move ions against their concentration gradient, which is the definition of active transport.

Specific activity is a measure of: A) The total units of an enzyme in a solution. B) The amount of substrate converted to product in one minute. C) The enzyme units per milligram of total protein in a solution. D) The concentration of enzyme that results in a particular rate of catalysis.

C - The answer is C because the activity is a measure of the amount of enzyme per milligram of total protein. This provides a measure of the purity of an enzymatic mixture.

Dendrotoxin from the mamba snake blocks voltage-gated potassium channels in somatic motor neurons that regulate skeletal muscle contraction. In what way would initial exposure to dendrotoxin affect the ability of a somatic motor neuron to propagate an electrical signal in response to a stimulus? A) It would inhibit the initiation of an action potential. B) It would shorten the refractory period. C) It would prolong the action potential. D) It would prevent depolarization.

C - The answer to this question is C because if potassium ion channels are blocked, the membrane would fail to repolarize, extending the length of the action potential and simulating excessive muscle contractions

Where in the human male reproductive system do the gametes become motile and capable of fertilization? A) Testis B) Urethra C) Epididymis D) Prostate gland

C - The answer to this question is C because sperm, produced in the seminiferous tubules of the testes, completes maturation and becomes motile in the epididymis.

The most likely reason that researchers added succinate was to stimulate: A) NADH synthesis by Complex I. B) GTP synthesis by succinyl-CoA synthetase. C) FADH2 synthesis by Complex II. D) Ubiquinol synthesis by fumarase.

C - The electron transport chain (ETC) depends on the production of NADH and FADH2 by the citric acid cycle and other metabolic pathways. Complex II is a flavoprotein (an oxidoreductase that contains an FAD or FMN prosthetic group) that is also known as succinate dehydrogenase. It is part of both the ETC and the citric acid cycle. In Complex II, electrons are transferred from succinate to an FAD molecule that is covalently attached to the protein complex, producing fumarate and FADH2. The FADH2 then transfers its electrons to ubiquinone, forming ubiquinol and regenerating FAD. Oxygen is the final electron acceptor in the ETC and is consumed only if sufficient NADH and FADH2 are present. Increasing the rate of FADH2 synthesis increases the rate of electron transfer and therefore increases the rate of oxygen consumption. In Figure 2, researchers measured oxygen consumption to assess ETC output under various conditions. They most likely added succinate to stimulate FADH2 synthesis (and subsequent electron transfer) by Complex II.

Which of the following is generally true about the path of blood flow through the heart? A) The right ventricle contracts to propel blood to all body tissues except the lungs. B) The left ventricle pumps blood into the left atrium. C) The aorta receives blood from the left ventricle. D) The right atrium receives blood from the pulmonary veins.

C - The heart is composed of two separate pumps that contract simultaneously; the right side of the heart pumps blood to the lungs, and the left side of the heart pumps blood to the rest of the body. The heart has four chambers: The left and right atria are receiving chambers, whereas the left and right ventricles are pumping chambers that supply the force necessary to propel blood through the circulatory system. The atrioventricular (AV) and semilunar (SL) valves prevent the backward flow of blood into the atria and ventricles, respectively. Accordingly, blood flows through the circulatory system as follows: 1. Deoxygenated blood from body tissues enters the right atrium via the (superior and inferior) vena cavae (Choice D). 2. The right atrium contracts, forcing the deoxygenated blood through the tricuspid valve and into the right ventricle. 3. The right ventricle contracts to propel the blood through the pulmonary valve and into the pulmonary arteries (Choice A). 4. The pulmonary arteries carry blood from the heart to the capillary beds of lung alveoli, where gas exchange occurs. 5. In pulmonary capillary beds, the blood unloads carbon dioxide and loads oxygen (ie, becomes oxygenated) and returns to the left atrium via the pulmonary veins. 6. The left atrium contracts, pushing the oxygenated blood through the bicuspid (mitral) valve and into the left ventricle (Choice B). 7. The left ventricle contracts to pump blood through the aortic valve to the rest of the body via the aorta. 8. The blood flowing through systemic blood vessels delivers nutrients and oxygen to the body tissues before returning to the heart.

The mineral component of human bone is a salt that consists primarily of all of the following EXCEPT: A) Calcium. B) Phosphate. C) Potassium. D) Hydroxyl groups.

C - The inorganic component of bone consists of submicroscopic deposits of calcium phosphate similar to hydroxyapatite (Ca10[PO4]6[OH]2), so it would be expected to contain calcium, phosphate and hydroxyl groups. Therefore, the correct answer states that the component not found in the salt is potassium, answer choice C.

The pKa for the side chain of cysteine is 8.4. What is the ratio of protonated side chain (-SH) to deprotonated side chain (-S−) in a solution of cysteine at physiological pH of 7.4? A) 1:100 B) 1:10 C) 10:1 D) 100:1

C - The pKa of a chemical group is the pH at which the ratio of protonated to deprotonated species is 1:1. A decrease in pH yields increased H+ concentration (favors protonation) whereas an increase in pH yields decreased H+ concentration (favors deprotonation). The protonated:deprotonated ratio of a substance can be determined by solving the Henderson-Hasselbalch equation. However, when the pH differs from the pKa by an integer number of pH units, the ratio can be determined more easily because each pH unit alters the ratio by a factor of 10.

Which two enzymes of the pentose phosphate pathway catalyze production of NADPH? A) Ribulose 5-phosphate epimerase and ribose 5-phosphate isomerase B) Transketolase and transaldolase C) Glucose 6-phosphate dehydrogenase and 6-phosphogluconate dehydrogenase D) Lactonase and aldolase

C - The pentose phosphate pathway generates ribose 5-phosphate and NADPH. The NADPH is generated by reduction of NADP+, which is catalyzed by the oxidoreductases glucose 6-phosphate dehydrogenase and 6-phosphogluconate dehydrogenase.

During muscle contraction, the power stroke is the pivoting motion of the myosin head that pulls the actin filament inward to shorten the sarcomere. Given this, which of the following events immediately precedes the power stroke? A) The binding of ATP to the myosin heads B) The hydrolysis of myosin-bound ATP to ADP and Pi C) The release of ADP and Pi from the myosin head D) The binding of Ca2+ to tropomyosin

C - When ATP bound to a myosin head hydrolyzes, the myosin head shifts back and attaches to the actin filament. Dissociation of ADP and Pi leads to a power stroke, which shortens the sarcomere. A new ATP molecule binds to the myosin head, causing it to dissociate from the actin filament. The action of many myosin heads cycling through this process leads to muscle contraction.

A prion is best described as an infectious: A) Prokaryote. B) Transposon. C) Protein. D) Virus.

C - because a prion is an abnormally folded protein that induces a normally folded version of the protein to also adopt the abnormal structure, which is often deleterious.

To confirm that β-oxidation is NOT impaired in healthy controls, researchers most likely measured which metabolic step in PBMCs? A) The exportation of citrate to the cytoplasm to produce acetyl-CoA B) The release of CO2 from malonyl-CoA C) The translocation of acylcarnitine to the mitochondrial matrix D) The depletion of reduced electron carriers

C - β-oxidation is the breakdown of fatty acid chains into acetyl-CoA through a series of oxidation reactions in the mitochondrial matrix that produce ATP. Short- and medium-chain fatty acids can diffuse through the inner and outer membranes of the mitochondria, but long-chain fatty acids obtained from lipid stores must undergo a set of reactions to enter the mitochondrial matrix. First, fatty acids in the cytoplasm are activated when the enzyme acyl synthetase attaches them to the carrier molecule CoA using ATP. Subsequently, in the rate-limiting step of the reaction, carnitine palmitoyltransferase I (CPTI) converts fatty acyl-CoA molecules into fatty acylcarnitine, which enters the intermembrane space. Fatty acylcarnitine is then moved by acylcarnitine translocase across the inner membrane and into the matrix. Lastly, to begin the oxidation reactions, carnitine palmitoyltransferase II (CPTII) on the inner membrane reconverts fatty acylcarnitine into fatty acyl-CoA, which can be broken down into acetyl-CoA. Therefore, researchers interested in confirming that fatty acid oxidation is not impaired would need to ensure that translocation of acylcarnitine into the mitochondrial matrix is functional.

Neural Crest

Cell migration in embryogenesis is the movement of cells into their final positions within the embryo. The migratory action of neural crest cells during neurulation (the formation of the nervous system) gives rise to many peripheral nervous system structures. In contrast, the central nervous system is derived from the neural tube. Neurulation is the formation of the nervous system in vertebrate organisms. Following gastrulation (formation of the three germ layers), the notochord, a mesodermal cylindrical structure, releases signals promoting the ectoderm above to thicken and form the neural plate. The neural plate folds inward, forming the neural groove, and the edges of the plate (the neural folds) converge to create the neural tube. The neural tube, which is the precursor of the central nervous system (CNS: brain and spinal cord), then pinches off the ectoderm. The remaining sections of the neural folds not included in the neural tube comprise the neural crest. Neural crest cells migrate away from the tube and toward the periphery of the embryo to give rise to most of the peripheral nervous system (PNS). Note that although somatic motor neurons are part of the PNS, their cell bodies originate in the CNS.

Deoxyribonucleotides, the building blocks of DNA, are each composed of a phosphate, a deoxyribose sugar, and one of four nitrogenous bases (A, T, C, or G).

Cytosine (C) and thymine (T) are one-ringed bases known as pyrimidines. Thymine contains a methyl group (-CH3) attached to the carbon atom in the fifth position, a feature that helps distinguish it from cytosine. Guanine (G) and adenine (A) are two-ringed bases known as purines. Guanine has a carbonyl group (-C=O) at the sixth position that helps differentiate it from adenine. In double-stranded genetic material, complementary base pairing is the bonding of a particular nucleotide with only one other type of nucleotide. Such base pairing is determined by the alignment of the hydrogen bond donors and acceptors of each nucleotide's nitrogenous base. Cytosine and guanine always base pair via three hydrogen bonds. The caption for Figure 1 states that the T291-1 mutation in ANGPTL4 results after substituting a cytosine with its complementary base (guanine), which is depicted in Choice B.

As it matures, the endosomal lumen becomes acidified relative to the cytosol. Protons from the cytosol most likely enter the endosomal lumen by which of the following processes? A) Simple diffusion B) Facilitated diffusion C) Passive transport D) Active transport

D - Acidification of the endosomal lumen as it matures from an early endosome to a late endosome involves the transport of protons against their concentration gradient from an area of relatively low concentration (cytosol; pH ≈ 7.4) to one of relatively high concentration (endosomal lumen; pH ≈ 5.5). Therefore, protons from the cytosol can only enter the endosomal lumen if energy is added to the system.

Assume bacteria are infected with a vector containing HBV genetic material. This vector has been shown to introduce its genomic content into bacterial cells without fully gaining entry into the bacterial cytoplasm. Based on this information, the vector is most likely a: A) Prion. B) Bacterium. C) Viroid. D) Bacteriophage.

D - Bacteriophages are viruses that exclusively infect bacteria but do not enter host cells to replicate their genetic material. Instead, they use their tail sheath, a structure that injects the phage genome into a bacterium. The remaining viral structures of the phage, such as the tail fibers, the capsid, and the tail sheath, are left outside the bacterium. Prion is a misfolded protein that acts as an infectious agent by inducing other normal proteins to change their secondary structure and become misfolded. These less soluble misfolded proteins aggregate and can cause disease. Prions do not contain genetic material and cannot transform bacteria. (Choice C) Viroids are not viruses; instead, they are pathogenic, circular, single-stranded RNA molecules lacking protein coats and primarily affect plants. They typically silence the expression of specific genes and inhibit protein synthesis by binding RNA sequences. Viroids enter cells by hiding inside viruses or through damaged tissue; they do not use the mechanisms described in the question.

Which step(s) of the viral life cycle would most likely occur in lytic phages? I Injection of viral genetic material into the bacterial host II Integration of viral DNA into the bacterial genome III Degradation of the host genome A) I only B) III only C) I and II only D) I and III only

D - Bacteriophages use the host cell's machinery and resources to replicate their genome and synthesize viral proteins to form new virions. Phages with a lytic life cycle replicate rapidly and release progeny via lysis of the host cell. In contrast, phages with a lysogenic life cycle integrate their genome with the host genome and replicate as the cell divides.

ADC isolated from the Gram-negative bacteria Alcaligenes faecalis showed a sigmoidal dependence on L-aspartate concentration. Which of the following could be the Hill coefficient for this form of ADC? A) 0 B) 0.5 C) 1 D) 2

D - Enzymes that exhibit increased binding affinity when the first subunit binds its substrate are described as positively cooperative (n > 1); these enzymes display sigmoidal (s-shaped) dependence on substrate concentration. In contrast, some enzymes show decreased binding affinity after the first subunit binds its substrate. These enzymes are said to be negatively cooperative (n < 1) and show hyperbolic dependence on substrate concentration. In addition, many enzymes exhibit no cooperativity (n = 1) and also display hyperbolic dependence on substrate concentration. The ADC obtained from Alcaligenes faecalis displays sigmoidal dependence on the concentration of its substrate, L-aspartate. Therefore, it exhibits positive cooperativity and must have a Hill coefficient that is greater than 1.

Which step is NOT required to generate the western blot shown in Figure 3? A) Gel electrophoresis of BAZ2B proteins recovered from affinity columns B) Transfer of proteins to a protein-binding membrane C) Blocking of nonspecific antibody binding D) Incubation with antibodies against histone subunit

D - H3A western blot is a technique used to detect the presence specific proteins. It is performed by gel electrophoresis of the protein of interest, followed by transfer to a protein-binding membrane (eg, nitrocellulose) and incubation with antibodies specific to the protein of interest. The primary antibody or a labeled secondary antibody is then detected by fluorescence or chemiluminescence.

Based on the passage that SIRT4 is an autosomally encoded sirtuin protein that colocalizes with the pyruvate dehydrogenase complex, the SIRT4 protein most likely contains which of the following? A) A nuclear localization sequence B) A polyubiquitin tag C) An N-linked carbohydrate chain D) A mitochondrial target signal

D - In eukaryotes, different metabolic processes are carried out in different compartments of the cell. For example, glycolysis and the pentose phosphate pathway occur in the cytosol, whereas the citric acid cycle and fatty acid oxidation occur in the mitochondria. The enzymes required to facilitate each process must be localized to the appropriate compartments. Transportation of proteins synthesized in the cytosol is prompted by a short amino acid sequence at the N-terminus, which serves as a signal for cellular machinery to move the protein to its correct destination. The pyruvate dehydrogenase complex (PDHC) catalyzes the decarboxylation of pyruvate to form acetyl-CoA, which then enters the citric acid cycle. This process occurs in the mitochondrial matrix, so the PDHC is found in this organelle. According to the passage, SIRT4 colocalizes with the PDHC and must also be found in the mitochondrial matrix. Therefore, SIRT4 most likely has a mitochondrial targeting signal to induce its transport to the mitochondria.

Researchers are studying a single-celled pathogen. Which experimental method would be LEAST effective for determining whether the pathogen is prokaryotic or eukaryotic? A) Analyzing the cells for the number of origins of replication B) Exposing the pathogen to a fluorescent antimitochondrial antibody C) Testing the pathogen for ribosome size D) Assessing the pathogen for the presence of pyruvate in the cytoplasm

D - In the given scenario, scientists must identify a characteristic unique to either prokaryotes or eukaryotes that will allow them to classify the pathogen into one of the two groups. However, glycolysis is a process that occurs in the cytoplasm of both prokaryotic and eukaryotic cells, and results in the synthesis of pyruvate. Therefore, the presence of pyruvate in the pathogen's cytoplasm would not be a defining characteristic that would help determine whether it is prokaryotic or eukaryotic. Mitochondria are the sites of aerobic respiration in eukaryotic cells but are absent in prokaryotic cells. An antimitochondrial antibody will bind only in the presence of eukaryotic cells, which would allow researchers to determine that the cell is eukaryotic and not prokaryotic.

A scientist sequences the genomes of several species and analyzes their evolutionary relationships to a single common ancestor. The percentage of similarity between the genome of each species to the genome of this ancestor is calculated and shown in the table below. Genomic similarity to common ancestor Species A 95% Species B 88% Species C 35% Species D 22% Which species diverged from the common ancestor first? A) Species A B) Species B C) Species C D) Species D

D - In this scenario, the genome of Species D is least similar to that of the common ancestor compared with the other species. This means that Species D diverged from the common ancestor first and accumulated more mutations across a longer time, which led to a small percentage similarity of 22% with the common ancestor.

Structural lipids in the cell membrane can be derived directly from dietary sources or from lipase-mediated catabolism. Which statement correctly describes lipid processing during digestion? A) All lipids are emulsified, but waxes cannot be hydrolyzed. B) Phospholipids can be hydrolyzed, but only steroids and vitamins are emulsified. C) Triglycerides are hydrolyzed, but only prostaglandins can be emulsified. D) All lipids can be emulsified, and triglycerides and phospholipids are hydrolyzed.

D - Lipid processing begins in the small intestine (duodenum), where bile salts break down lipid globules into smaller droplets in a process called emulsification. This process results in the formation of spherical structures, known as micelles, composed of a hydrophobic core containing the nonpolar hydrocarbon tails of lipids and an outer shell of polar head groups that make contact with water. The formation of micelles increases the surface area of lipid available for hydrolysis by lipases. Hydrolyzable lipids contain ester bonds that can be cleaved by lipases through the addition of a water molecule (hydrolysis). These lipids include triacylglycerols, phospholipids, sphingolipids, and waxes. Nonhydrolyzable lipids do not contain the ester linkages necessary for lipase digestion. The most common nonhydrolyzable dietary lipids are cholesterol (steroids) and fat-soluble vitamins (A, D, E, and K). The question states that some dietary lipids are catabolized (broken down) by lipases, whereas others can be absorbed directly. The most accurate description of lipid processing acknowledges that all lipids are emulsified during digestion but only some lipids, including triglycerides and phospholipids, contain hydrolyzable ester bonds.

In order to transport long-chain fatty acids from the cytosol to the mitochondria, three of the following molecules are required. Which one is NOT necessary? A) Coenzyme A B) ATP C) Carnitine D) Oxaloacetate

D - Long-chain fatty acids are not recognized by any transport proteins; they must be modified or "activated" to enter the matrix. Fatty acid activation and transport includes the following steps: In the cytosol, the enzyme acyl-CoA synthetase catalyzes the reaction of fatty acids with coenzyme A to form acyl-CoA molecules. This reaction is thermodynamically unfavorable and requires ATP hydrolysis to proceed (Choices A and B). Acyl-CoA molecules then migrate to the intermembrane space, where they can react with carnitine to form acylcarnitine. The transport protein acylcarnitine translocase, located on the inner mitochondrial membrane, recognizes acylcarnitine and carries it into the mitochondrial matrix (Choice C). Acylcarnitine is then converted back to acyl-CoA and carnitine. Acyl-CoA is subsequently digested by β-oxidation, and carnitine is transported out of the matrix where it can pick up and carry a new fatty acid into the mitochondrial matrix.

Which of the following is true concerning the proteins to which AAV2 binds in Figure 1, Step 1 (transmembrane proteins)? A) They localize primarily in low-cholesterol environments known as lipid rafts. B) They are held statically in place by cytoskeletal components. C) They are able to interact only with the phospholipids on the outer leaflet of the cell membrane. D) They can migrate laterally through the phospholipid-rich environment of the cell membrane.

D - Many membrane proteins, called transmembrane proteins, span the width of the membrane. Figure 1 shows that AAV2 binds to transmembrane proteins, which must be able to migrate laterally through the phospholipid environment of the cell membrane. Although it is believed that transmembrane proteins often localize to relatively ordered portions of the membrane called lipid rafts, these rafts are high in cholesterol content. In addition, the passage gives no information about the structure of the AAV2 transmembrane receptor, so specifics of its location within the cell membrane cannot be determined. (Choice B) Cytoskeletal components such as actin filaments interact with some transmembrane proteins, but they do not hold these proteins statically in place. Therefore, these transmembrane proteins can move dynamically within the cell membrane. (Choice C) The phospholipid bilayer of the cell membrane has an outer leaflet that interacts with the extracellular environment and an inner leaflet that interacts with the cytoplasm of the cell. Transmembrane proteins cross the entire phospholipid bilayer and therefore interact with both the outer and inner leaflets. Proteins that interact with only one leaflet are not transmembrane proteins.

Phospholipases release arachidonic acid from cell membranes as part of the inflammatory response. Given this information, which type of molecule is derived from arachidonic acid and is involved in this response? A) Terpenes B) Ketone bodies C) Catecholamines D) Prostaglandins

D - Prostaglandins act as autocrine and paracrine signals (binding to receptors on or near the cell from which they originated). Because they act only on nearby cells, prostaglandins produce a localized inflammatory response. Nonsteroidal anti-inflammatory drugs (NSAIDs) such as aspirin and ibuprofen treat inflammation by inhibiting prostaglandin synthesis. Terpenes are precursors in the synthesis of cholesterol and steroid hormones. They are derived from isoprenes and do not play a role in inflammatory responses. Ketone bodies are fuel sources that are produced under low glucose conditions. They are derived from acetyl-CoA and are not involved in inflammatory responses. Catecholamines such as epinephrine and norepinephrine are hormones derived from the amino acid tyrosine. They can play a role in inflammatory responses but generally do so as endocrine signals (acting on cells that are distant from their origin).

Which experimental procedure(s) must scientists use to determine Vmax and Km of an enzymatic reaction using the Michaelis-Menten model? They must ensure that: I They only measure the initial reaction rate for each substrate concentration. II Total enzyme concentration is much greater than the Km of the reaction. III Each initial substrate concentration tested is much greater than enzyme concentration. A) I only B) III only C) I and II only D) I and III only

D - The Michaelis-Menten equation models the initial rates of a reaction at various substrate concentrations. Rates are measured as the slope of the initial, linear phase of the reaction before substrate is depleted and product accumulates. Enzyme concentration should be much lower than all substrate concentrations tested.

Which event is LEAST likely to occur during apoptosis? A) Cytochrome c release from the mitochondria B) Cell shrinkage and blebbing C) Caspase activation D) Damage to nearby cells caused by lysis

D - The answer is D because damage to nearby cells caused by lysis is a characteristic of necrosis, not apoptosis.

According to the cross-bridge model of muscle contraction, the muscles stiffen after death because ATP is unavailable to bind and directly release: A) ADP from the actin head. B) ADP from the myosin head. C) The actin head from the myosin filament. D) The myosin head from the actin filament.

D - The answer to this question is D because during normal muscle contraction, ATP is required to break the bonds between the actin filament and the myosin head. After death, no new ATP is generated, so the myosin head cannot be released from the actin filament, resulting in stiffening of muscles

After a section of a DNA strand containing a UVR-induced lesion is removed and resynthesized, the newly synthesized strand is rejoined to the remainder of the DNA strand by what type of bond? A) Disulfide B) Hydrogen C) Peptide D) Phosphodiester

D - The answer to this question is D because phosphodiester bonds link the 3ʹ carbon atom of one deoxyribose and the 5ʹ carbon atom of another deoxyribose within the DNA molecules.

Based on the passage, CatB or CatL (proteases) or both would be expected to have which of the following effects, if any, on EGP? A) No effect B) Reduction of enzyme activity C) Formation of protein dimers D) Digestion into smaller protein fragments

D - The answer to this question is D because the passage states that CatB and CatL are proteases. Proteases function to digest proteins into smaller fragments.

Which combination of lipid type and temperature results in the most disordered membrane? A) Lipids with saturated acyl chains at low temperature B) Lipids with saturated acyl chains at high temperature C) Lipids with unsaturated acyl chains at low temperature D) Lipids with unsaturated acyl chains at high temperature

D - The key is D because lipids with unsaturated acyl chains do not pack well against one another, increasing the level of disorder. Increased temperature also increases the level of disorder in the membrane because there is more movement at higher temperatures

Cirrhosis of the liver is characterized by hepatic cell death, tissue scarring, and altered liver function. All of the following would be disrupted by cirrhosis of the liver EXCEPT: A) The production of ketone bodies. B) The metabolism of most drugs. C) The oncotic pressure within capillaries. D) The secretion of digestive proteolytic enzymes.

D - The liver has various functions, including storage of glycogen, regulation of blood glucose levels, detoxification of drugs, synthesis and secretion of bile, production of serum proteins (eg, albumin, clotting factors), and synthesis of ketone bodies.

Albumin is the major blood osmoregulatory protein. The most likely effect of a sharp rise in the level of serum albumin is: A) A drop in blood pressure. B) An increase in immune response. C) An efflux of albumin into the interstitial fluids. D) An influx of interstitial fluid into the bloodstream.

D - The question asks the examinee to identify the most likely effect of a sharp rise in the level of serum albumen, a major blood osmoregulatory protein. A sharp rise in osmotically active albumin in the serum would increase the flow of interstitial fluid into the bloodstream and result in an increase in blood pressure, not a decrease. Therefore, A is incorrect, and D is correct. B is incorrect because the immune response does not depend on the level of serum albumen. C is incorrect because albumen would not normally pass through capillary walls. Thus, D is the best answer.

Researchers hypothesized that administration of canagliflozin reduces blood pressure in patients with diabetes by increasing urine output and reducing total blood volume. A decrease in which of the following would also result in lowered blood pressure? I. Activation of the renin-angiotensin system II. Production of aldosterone III. Secretion of antidiuretic hormone A) II only B) III only C) I and III only D) I, II, and III

D - The renin-angiotensin system (RAS) is a multi-organ molecular cascade activated when BP (or blood volume) falls. A drop in BP causes the juxtaglomerular cells in the kidneys to release renin, an enzyme that cleaves the plasma protein angiotensinogen to form angiotensin I. Angiotensin-converting enzyme (ACE) then cleaves angiotensin I to form angiotensin II. Angiotensin II ultimately raises BP by inducing both the release of aldosterone from the adrenal cortex (increasing BP by increasing blood volume through water retention) and the constriction of arterioles (increasing BP without changing blood volume) (Number I). Aldosterone is released in response to RAS activation or to an increased serum level of K+. Aldosterone acts on the distal tubules and collecting ducts of nephrons to promote the reabsorption of Na+ and the secretion of K+. Increased reabsorption of Na+ increases the osmolarity, or solute concentration, of the renal interstitial fluid. Elevated osmolarity promotes water reabsorption, which ultimately causes blood volume and BP to increase (Number II). ADH is released by the posterior pituitary when BP falls or when blood osmolarity rises. ADH promotes water reabsorption by increasing the permeability of the distal tubule and collecting duct to water. ADH also induces vasoconstriction, the narrowing of blood vessels. Both of these effects increase BP (Number III). Angiotensin II of the RAS pathway, aldosterone, and ADH are hormones that cause blood pressure to increase. Accordingly, a decrease in RAS activation, ADH secretion, or aldosterone production would cause BP to fall.

Which type of molecule, primarily obtained from the diet, would most likely accumulate in adipose tissue? A) Steroid hormones B) Water-soluble vitamins C) Peptide hormones D) Fat-soluble vitamins

D - Vitamins are classified as either water-soluble (B series, C) or fat-soluble (A, D, E, K). Water-soluble vitamins contain several hydrophilic groups and are easily dissolved in aqueous environments. They are not stored in the body, and the excess is excreted in the urine (Choice B). For this reason, water-soluble vitamins must be frequently replenished. In contrast, fat-soluble vitamins are highly hydrophobic. They dissolve in lipid rather than aqueous environments and cannot be easily excreted in urine. Instead, the excess is stored and accumulates in adipose tissue, which consists mainly of fat cells.

Hormones secreted by the adrenal medulla most likely influence energy metabolism by: A) Inhibiting proteins that mediate glycogen synthesis. B) Activating their cytosolic receptors in target cells. C) Reducing the activity of enzymes that catalyze fat hydrolysis. D) Increasing the activity of proteins that inhibit gluconeogenesis.

Energy metabolism refers to the processes by which the body manages cellular energy stores. The adrenal gland increases the body-wide level of free cellular energy sources by releasing glucocorticoids, norepinephrine, and epinephrine. Norepinephrine and epinephrine are released by the adrenal medulla to modulate the metabolism of glycogen, the storage form of glucose. In addition, both hormones promote the breakdown of glycogen into glucose monomers in a process known as glycogenolysis. By inhibiting the enzymes that mediate glycogen synthesis, norepinephrine and epinephrine also inhibit glycogenesis (the formation of glycogen from glucose).

Which statement most accurately describes the role of T tubules in skeletal muscle cells? A) T tubules bind acetylcholine at the neuromuscular junction to generate a depolarizing stimulus. B) Depolarizing current reaches the sarcoplasmic reticulum by traveling down T tubules. C) Muscle contraction is driven by the sliding of T tubules across one another in the sarcomere. D) T tubules sequester Ca2+ out of the cytosol to prevent prolonged muscle contraction.

For a skeletal muscle cell to contract, Ca2+ must be released into the cytosol from the sarcoplasmic reticulum (SR). Ca2+ release is induced when a depolarizing current (action potential) runs along the sarcolemma and travels down the T tubules. This current causes the nearby SR to open its Ca2+ channels, allowing Ca2+ ions to flow into the cytosol and induce the sarcomeric actin-myosin interactions required for muscle contraction.

Amino Acid Properties

Hydrophilic amino acids may be polar but uncharged (S, T, C, N, Q, Y), acidic (negatively charged at pH 7.4; D, E), or basic (positively charged at pH 7.4; K, R, H). Note that although tyrosine (Y) has a polar OH group, it is often classified as hydrophobic. Hydrophobic amino acids may be aliphatic (linear hydrocarbons; G, A, P, V, L, I, M) or aromatic (benzene ring forms; F, W). Note that tyrosine and histidine, which are polar, also contain aromatic rings.

Fatty Acid Metabolism Regulation

Liver cells respond to insulin by increasing their rates of lipid synthesis and mobilization. In the mitochondria, insulin activates pyruvate dehydrogenase (PDH), which converts pyruvate into acetyl-CoA. The accumulated acetyl-CoA is transported to the cytoplasm by combining with oxaloacetate to form citrate, which can exit the mitochondria. In the cytoplasm, citrate is converted back to acetyl-CoA and oxaloacetate. Insulin activates both acetyl-CoA carboxylase (ACC) and fatty acid synthase (FAS), which convert acetyl-CoA to malonyl-CoA and catalyze the synthesis of 16-carbon fatty acid chains from malonyl-CoA, respectively.

DNA Curve Melting Analysis

Melted DNA strands will reanneal to form the double helix once temperature decreases, but the time it takes for complementary strands to reanneal depends on the following: DNA length: Longer DNA molecules have more hydrogen bonds and will take more time to both melt and reanneal. pH: The physiological pH range (7.3-7.4) allows maximal hydrogen bonding between nitrogenous bases of DNA. At low pH, hydrogen bond acceptor atoms in the bases become protonated, and these protonated acceptors cannot form hydrogen bonds, causing double helix separation. In contrast, high pH causes deprotonation of hydrogen bond donors in the bases; the loss of protons results in the loss of hydrogen bonds and a destabilized double helix. Salt concentration (ionic strength): The electrostatic repulsion between negatively charged phosphate groups on the sugar-phosphate backbone destabilizes the double helix. However, this repulsion is neutralized and shielded by the binding of positively charged species in solution (eg, Na+ and Mg2+ cations). High salt concentration of the solution increases double helix stability, but low salt concentration decreases stability.

Cell Membrane Fluidity

Short tails participate in fewer hydrophobic interactions than long tails and therefore result in increased fluidity. Carbon-carbon double bonds in fatty acyl tails are typically in the cis-conformation and introduce a bend in the tail, preventing adjacent tails from interacting efficiently with each other. A higher number of double bonds leads to decreased interaction and greater fluidity.

Which of the following would most likely cause an embryo to implant in a location other than the uterine lining? A) Reduced number of fallopian cilia B) Surge in luteinizing hormone prior to ovulation C) Incomplete gastrulation D) Lack of follicle rupture from the ovary

A - It is normal for fertilization to occur in the fallopian tube prior to implantation within the uterus. Typically, small hair-like structures within the fallopian tube called fallopian cilia help propel the fertilized oocyte toward the uterus for implantation. However, without an adequate number of cilia in the fallopian tube, improper implantation of the fertilized egg can occur outside the uterus.

In the absence of Frizzled activation, β-catenin is covalently modified by being targeted for ubiquination and: A) Bound by a proteasome to initiate degradation into short peptides. B) Translocated into the Golgi body for secretion through exocytosis. C) Engulfed by a lysosome where it is hydrolyzed by proteases. D) Stored in vesicles until the signaling pathway is activated.

A - . Ubiquitination targets a protein for degradation by a proteasome

During gluconeogenesis, the enzyme phosphoenolpyruvate carboxykinase (PEPCK) catalyzes production of phosphoenolpyruvate (PEP) by decarboxylating oxaloacetate and transferring a phosphate group from GTP. What is the balanced equation for this reaction? A) Oxaloacetate + GTP → GDP + CO2 + PEP B) Oxaloacetate + GTP + PEPCK → GDP + 2 CO2 + PEP C) Oxaloacetate + GTP → GDP + 2 Pi + PEP D) Oxaloacetate + GTP + PEPCK → GDP + Pi + PEP

A - According to the question, phosphoenolpyruvate carboxykinase (PEPCK) catalyzes the formation of phosphoenolpyruvate (PEP) during gluconeogenesis through the following steps (note that reactants always appear on the left side of the equation whereas products appear on the right): During decarboxylation, CO2 is removed from oxaloacetate (a reactant), yielding CO2 as a product of the reaction. GTP (a reactant) transfers a phosphate group to the decarboxylated oxaloacetate, producing GDP and PEP. Because PEPCK is an enzyme, it facilitates these steps but is not altered by the reaction. Enzymes may be written above or below a reaction arrow to show their participation, but they never appear on the reactant side (left) or product side (right) of the arrow.

Which of the following statements best describes a BCB+ oocyte that is arrested in metaphase II? A) It cannot continue cell division without fertilization. B) It cannot complete meiosis without losing the first polar body. C) It is unable to be penetrated by sperm. D) It is no longer susceptible to nondisjunction.

A - The secondary oocyte remains frozen in metaphase II until fertilization occurs, at which point it completes its second meiotic division (telophase II) into an ootid and second polar body. The zygote that results from fertilization is diploid due to genetic contribution from both the ovum and sperm nuclei.

Fatty Acid Oxidation

All fatty acids are broken down into multiple two-carbon units called acetyl-CoA. Fatty acids with an even number of carbons yield half as many acetyl-CoA units as the total number of carbons in the original molecule. If the fatty acid has an odd number of carbons, the final three carbons are converted into a compound called propionyl-CoA. In other words, for odd-chain fatty acids, the number of acetyl-CoA units produced equals (total carbons − 3)/2. The question states that the β-oxidation resulted in 9 acetyl-CoA units, so the original fatty acid must have had twice that number of carbons (18). The fact that no isomerization reactions were needed indicates that the fatty acid was saturated.

Individuals with beta-thalassemia are at risk for developing an enlarged spleen. The resulting impairment of spleen function in these individuals would NOT affect: A) Immune responses against viral or bacterial infection. B) Regulation of blood glucose concentration. C) Removal of aged red blood cells from the circulation. D) Storage of red blood cells.

B - The spleen is involved in immune function, blood storage, and the filtration and removal of senescent red blood cells from the blood.

Which of the following would NOT be present in the structure of the dFTP nucleotide (base pair analog) used for replication? A) Bond to phosphate on the 5′ carbon of the pentose sugar B) Glycosidic bond on the 1′ carbon of the pentose sugar C) OH group on the 2′ carbon of the pentose sugar D) H group on the 5′ carbon of the pentose sugar

C - A nucleoside is a pentose (five-carbon) sugar linked to a nitrogenous base on the 1′ carbon by a covalent glycosidic bond. Nucleotides consist of a nucleoside attached to one or more phosphate groups by a phosphoester bond. If the pentose has a hydroxyl group at the 2′ carbon, it is ribose; if a hydrogen is present at the same position, it is deoxyribose.

Human sperm and oocytes are substantially different in all of the following aspects EXCEPT: A) Cell volume. B) Maturation stage at birth. C) The contribution of chromosomes to a zygote. D) The rate at which they are produced.

C - Egg and sperm cells are similar in ploidy as they both are haploid cells and contribute the same number of chromosomes to a zygote.

The amino acid precursor of serotonin is best described as having which type of R group? A) Nonpolar, aliphatic B) Polar, uncharged C) Aromatic D) Negatively charge

C - Tryptophan, the amino acid precursor of serotonin, contains an aromatic R group.

Researchers treated 5/6 Nx mice with regular intraperitoneal infusions of CaCl2 to cause a sustained increase in serum calcium levels. The hormone that was released as a result of this condition was monitored using a hormone-specific radiolabeled antibody. In the tested animals, what organ would most likely show increased binding of radiolabeled antibody? A) Thymus B) Pancreas C) Bone marrow D) Thyroid

D - In response to increased plasma calcium concentration, the peptide hormone calcitonin is secreted by the thyroid glands.

Evolution

Convergent evolution leads to similar characteristics in distantly related species that are exposed to similar environmental pressures. Parallel evolution also leads to similar characteristics in species found in similar environments but occurs in species with a more recent common ancestor. Divergent evolution leads to unique characteristics in somewhat closely related species that face contrasting environmental pressures.

During periods of stress, many individuals experience dry mouth caused by decreased salivation. Which of the following functions would also be expected to decrease during periods of stress? A) Gluconeogenesis B) Cardiac contraction C) Dilation of the pupils D) Digestive peristalsis

D - Sympathetic activation would be expected to increase gluconeogenesis, cardiac contractility, and pupil dilation (diameter).

Protein Structure

Protein structure is organized into primary, secondary, tertiary, and quaternary levels. Primary structure is the sequence of amino acids linked together by peptide bonds. Secondary structure consists of local structures such as α-helices and β-sheets stabilized by backbone hydrogen bonds. Tertiary and quaternary structures describe the three-dimensional form of a protein, mediated by hydrophobic, ionic, and hydrogen bond interactions. Because reflectins do not fold, they only participate in primary structure, characterized by the covalent carbon-nitrogen linkages of peptide bonds.

GI Cells

The epithelial cells that line the gastrointestinal tract are typically highly proliferative

Active Transport

There are two types of active transport: Primary (direct) active transport uses the energy released by ATP hydrolysis (performed by a transmembrane ATPase). Secondary (indirect) active transport is a coupled transport process that utilizes the energy released by the movement of one substance along its concentration gradient (passive transport) to move another substance against its concentration gradient. Essentially, the potential energy stored in the concentration gradient of one molecule is used for the transport of another molecule.


संबंधित स्टडी सेट्स

Oedipus Rex Part II Questions and Literary Analysis

View Set

QCAA Unit 4 annotated bibliography Psychology

View Set

115 Midterm II Pavlovian Eye-Blink Conditioning & Cerebellar Motor Learning

View Set

AP HUG Unit 5 StudyGuide Quiz Questions

View Set

Science Trivia 7.0 (Scientist & their Discoveries)

View Set